Archivi categoria: Esame di maturità scientifica svolto

Esame di maturità 2024 prova di matematica – Prova svolta

Indice

Problema 1

Punto a

Per prima cosa calcoliamo la derivata della funzione f_{a,b}(x) che è

    \[f'_{a,b}(x)={3ax^2\cdot x^2-(ax^3+b)\cdot 2x\over x^4}={ax^4-2bx\over x^4}\]

a questo punto se vogliamo che la curva sia tangente alla retta t nel punto di ascissa x=1 sicuramente dobbiamo avere che la curva e la retta passino dallo stesso punto per x=1, ossia P(1\:;\:5) e che la derivata in quel punto sia f'_{a,b}(1)=-7. Pertanto

    \[f_{a,b}(1)=5\Rightarrow a+b=5\]

    \[f'_{a,b}(1)=-7\Rightarrow a-2b=-7\]

da cui b=4 e a=1.

Ritorna all’indice

Punto b

Studiamo ora la funzione f(x)={x^3+4\over x^2}. Sappiamo che il suo dominio è tutto l’insieme dei numeri reali meno lo 0. Vediamo ora i limiti nei punti estremi del dominio

    \[\lim_{x\to -\infty}{x^3+4\over x^2}=-\infty\]

    \[\lim_{x\to +\infty}{x^3+4\over x^2}=+\infty\]

Cerchiamo un possibile asintoto obliquo

    \[m=lim_{x\to \infty}{f(x)\over x}=lim_{x\to \infty}{x^3+4\over x^3}=1\]

    \[q=lim_{x\to \infty}f(x)-mx=lim_{x\to \infty}{x^3+4\over x^2}-x=0\]

per cui abbiamo un’asintoto obliquo di equazione y=x. Cerchiamo ora un possibile asintoto verticale facendo i limiti per x che tende a 0

    \[\lim_{x\to 0^-}{x^3+4\over x^2}=+\infty\]

    \[\lim_{x\to 0^+}{x^3+4\over x^2}=+\infty\]

Cerchiamo infine i punti di massimo e minimo

    \[f'(x)=0\Rightarrow x=2\]

da cui possiamo disegnare la funzione

Completiamo ora il punto b) scrivendo l’equazione dell’ulteriore retta tangente alla curva e passante per P, osserviamo anche che tale retta non sarà tangente nel punto P, ma semplicemente passerà per tale punto. La retta tangente in un generico punto A=(a\:;\:{a^3+4\over a^2}) con a\neq 0 ha equazione

    \[y-{a^3+4\over a^2}=(1-{8\over a^2})(x-a)\]

pertanto, imponendo il passaggio per il punto P otteniamo

    \[5-{a^3+4\over a^2}=(1-{8\over a^2})(1-a)\]

da cui si ottiene

    \[a^3-3a+2=0\Rightarrow (a-1)^2(a+2)=0\Rightarrow a=-2\:e\:a=1\]

pertanto la seconda retta tangente passa, ed è tangente, al punto Q(-2\:;\:-1) ed ha equazione

    \[y+1=2(x+2)\Rightarrow y=2x+3\]

Ritorna all’indice

Punto c

Per risolvere questo punto mettiamo a sistema l’equazione del fascio y-5=m(x-1) e l’equazione della curva. Risolviamo per sostituzione sostituendo y=5+m(x-1) e otteniamo

    \[5+m(x-1)={x^3+4\over x^2}\]

    \[5x^2+mx^3-mx^2=x^3+4\]

    \[(m-1)x^3+(5-m)x^2-4=0\]

Ragioniamo ora sulle soluzioni di quell’equazione di terzo grado. Vediamo subito che per m=1 l’equazione diventa l’equazione di secondo grado x^2-1=0, pertanto abbiamo due soluzioni. Per m<1 il coefficiente direttore m-1 è negativo e siccome tale polinomio passa dal punto (0\:;\:-4) ne consegue che deve necessariamente avere tre soluzioni, pertanto per m<1 le soluzioni sono tre. Con 1<m<2 si hanno tre soluzioni distinte. Mentre quando m=2 l’equazione di terzo grado x^3+3x^2-4=0 ha due soluzioni coincidenti e quando m diventa maggiore di 2 si ha un’unica soluzione reale.

Ritorna all’indice

Punto d

Quello che ci viene chiesto di calcolare è l’area della regione arancione presente in questo grafico man mano che la retta di equazione x=k si sposta sempre più a destra

geometricamente possiamo subito osservare che tale limite dovrà tendere a un numero proprio per definizione di asintoto obliquo. Vediamo questo dal punto di vista analitico. L’area della figura arancione la possiamo calcolare facendo

    \[\lim_{k\to +\infty}(\int_1^{3\over 2}{x^3+4\over x^2}-(-7x+12)dx+\int_{3\over 2}^k {x^3+4\over x^2}-xdx)\]

    \[\lim_{k\to +\infty}(\int_1^{3\over 2}8x+{4\over x^2}-12dx+\int_{3\over 2}^k {4\over x^2}dx)\]

    \[\lim_{k\to +\infty}(4\cdot \Big[x^2-{1\over x}-3x\Big]_1^{3\over 2}+{8\over 3}-{4\over k})\]

    \[\lim_{k\to +\infty}({1\over 3}+{8\over 3}-{4\over k})=3\]

Ritorna all’indice

Problema 2

Punto a

Per verificare che la famiglia di funzioni f_n(x) non è derivabile nel punto di ascissa x=0 calcoliamo il limite del rapporto incrementale da destra e da sinistra e verifichiamo come si comporta. Innanzitutto abbiamo che

    \[f_n(x)=\sqrt[n]{x^2}-\sqrt{ax^2+bx+1}=x^{2\over n}-(ax^2+bx+1)^{1\over 2}\]

pertanto in un intorno di x=0 la funzione è perfettamente definita e quindi possiamo dire che

    \[f'_n(x)={2\over n}\cdot x^{{2\over n}-1}-{1\over 2}\cdot (ax^2+bx+1)^{{1\over 2}-1}\cdot (2ax+b)\]

A questo punto separiamo i due casi n=2 e n>2. Nel primo caso, con n=2, la funzione diventa

    \[f_2(x)=\left | x \right |-\sqrt{ax^2+bx+1}\]

che quindi derivata diventa per x<0

    \[f'_2(x)=-1-{1\over 2}\cdot (ax^2+bx+1)^{-{1\over 2}}\cdot (2ax+b)\]

e per x>0

    \[f'_2(x)=1-{1\over 2}\cdot (ax^2+bx+1)^{-{1\over 2}}\cdot (2ax+b)\]

quindi il limite destro e il limite sinistro sono diversi, infatti

    \[\lim_{x\to 0^-}f_2(x)=\lim_{x\to 0^-}-1-{1\over 2}\cdot (ax^2+bx+1)^{-{1\over 2}}\cdot (2ax+b)=-1-{b\over 2}\]

    \[\lim_{x\to 0^+}f_2(x)=\lim_{x\to 0^+}1-{1\over 2}\cdot (ax^2+bx+1)^{-{1\over 2}}\cdot (2ax+b)=1-{b\over 2}\]

Analizziamo ora il secondo caso, cioè n>2, in questa situazione il primo termine della derivata diventa

    \[{2\over n}\cdot x^{{2\over n}-1}={2\over nx^{n-2\over n}}\]

pertanto il limite sinistro viene

    \[\lim_{x\to 0^-}f_n(x)=\lim_{x\to 0^-}{2\over nx^{n-2\over n}}-{1\over 2}\cdot (ax^2+bx+1)^{-{1\over 2}}\cdot (2ax+b)=\pm\infty\]

dove il più lo abbiamo nel caso in cui n sia pari, mentre il segno meno nel caso in cui n sia dispari. Per il limite destro invece abbiamo il risultato sempre positivo, ossia

    \[\lim_{x\to 0^+}f_n(x)=\lim_{x\to 0^+}{2\over nx^{n-2\over n}}-{1\over 2}\cdot (ax^2+bx+1)^{-{1\over 2}}\cdot (2ax+b)=+\infty\]

Questo ragionamento ci fornisce anche la risposta ai punti angolosi, infatti tali punti li abbiamo quando i limiti sono diversi, pertanto quando n è un numero dispari oppure quando n=2.
Determiniamo ora i parametri a, b in maniera tale che il grafico sia quello in figura

Siccome deve essere simmetrico rispetto all’asse y abbiamo che

    \[f_2(x)=f_2(-x)\]

pertanto

    \[\left | x \right |-\sqrt{ax^2+bx+1}=\left | -x \right |-\sqrt{a(-x)^2-bx+1}\]

    \[\sqrt{ax^2+bx+1}=\sqrt{ax^2-bx+1}\]

per cui necessariamente b=0, per determinare il valore di a imponiamo che il dominio sia [-1\:;\:1], quindi risolviamo la disequazione

    \[ax^2+1\geq 0\Rightarrow x\in[-{1\over \sqrt{a}}\:;\:+{1\over \sqrt{a}}]\]

pertanto se vogliamo che quell’intervallo sia [-1\:;\:1] dobbiamo avere a=-1.

Ritorna all’indice

Punto b

Studiamo ora la funzione

    \[g(x)=\left | x \right |+\sqrt{1-x^2}\]

ancora una volta il suo dominio è la soluzione della disequazione 1-x^2\geq 0 pertanto è l’intervallo [-1\:;\:1]. Inoltre la sua derivata è

    \[g'(x)=\mp 1-{2x\over 2\sqrt{1-x^2}}\]

dove il segno meno lo abbiamo quando x<0 mentre il segno più quando x>0. Pertanto

    \[lim_{x\to -1^+}g'(x)=lim_{x\to -1^+} -1-{2x\over 2\sqrt{1-x^2}}=+\infty\]

    \[lim_{x\to +1^-}g'(x)=lim_{x\to +1^-} 1-{2x\over 2\sqrt{1-x^2}}=-\infty\]

quindi negli estremi del dominio la funzione non è derivabile, vediamo ora come si comporta per x=0.

    \[lim_{x\to 0^-}g'(x)=lim_{x\to 0^-} -1-{2x\over 2\sqrt{1-x^2}}=-1\]

    \[lim_{x\to 0^+}g'(x)=lim_{x\to 0^-} 1-{2x\over 2\sqrt{1-x^2}}=+1\]

pertanto neanche nel punto x=0 la funzione non è derivabile. Una volta verificato questo procediamo con lo studio di funzione, l’unica cosa che ci manca per poter disegnare la funzione g(x) è lo studio della derivata prima per poter determinare le concavità e i massimi e minimi, quindi studiamo g(x)>0. Per x<0 abbiamo

    \[-1-{2x\over 2\sqrt{1-x^2}}>0\Rightarrow x<-{1\over \sqrt 2}\]

quindi nel punto x=-{1\over \sqrt 2} abbiamo un massimo relativo. Per completare osserviamo che g(x) è una funzione pari, pertanto il suo grafico è simmetrico rispetto all’asse y. Una volta fatto questo disegniamo i due grafici uniti



Ritorna all’indice

Punto c

I punti di intersezione tra la retta x=k e la nostra curva a cuore sono le intersezioni tra le due curve con la retta (infatti nella parte bassa del cuore la curva blu è rappresentata dalla curva f(x) mentre nella parte alta del cuore la curva rossa è rappresentata dalla curva g(x)). Pertanto le coordinate y dei due punti di intersezione sono f(k) e g(k), osserviamo anche che siccome entrambe le funzioni sono pari a noi basta studiare la situazione per k\in[0\:;\:1]

    \[f(k)=k-\sqrt{1-k^2}\]

    \[g(k)=k+\sqrt{1-k^2}\]

quindi la formula che esprime la distanza tra questi due punti sarà

    \[g(k)-f(k)=k+\sqrt{1-k^2}-(k-\sqrt{1-k^2})=2\sqrt{1-k^2}\]

che è una funzione strettamente decrescente e quindi ha il massimo per k=0.

Ritorna all’indice

Punto d

Per verificare che H(x) è una primitiva di h(x) ci basta derivare H(x) e vedere che, a meno di una costante, viene h(x). Quindi

    \[H'(x)={1\over 2}({1\over \sqrt{1-x^2}}+1\cdot \sqrt{1-x^2}-x\cdot {2x\over 2\sqrt{1-x^2}})={1\over 2}({1+1-x^2-x^2\over \sqrt{1-x^2}})={1\over 2}({2-2x^2\over \sqrt{1-x^2}})=h(x)\]

Infine per calcolare l’area del cuore impostiamo i seguenti integrali

    \[A=2\cdot (\int_0^1(g(x)-f(x))dx)=2\int_0^12\sqrt{1-x^2}dx=4\cdot \Big[H(x)\Big]_0^1\]

    \[A=\Big [\arcsin x+x\sqrt{1-x^2}\Big]_0^1=2(\arcsin 1-\arcsin0)=\pi\]

Ritorna all’indice

Quesito 1

Partiamo con la prima parte della dimostrazione. Sia quindi ABC un triangolo rettangolo isoscele e vediamo che la sua altezza relativa all’ipotenusa deve essere la metà dell’ipotenusa.

Sicuramente l’angolo in C è un angolo di 45^\circ, siccome il triangolo grande è un triangolo isoscele e rettangolo, e sicuramente l’angolo BHC è un angolo retto siccome BH è altezza, ne segue che anche l’angolo HBC è di 45^\circ e pertanto il triangolo BCH è isoscele. Quindi possiamo concludere che la lunghezza di BH e la lunghezza di HC coincidono, ma sappiamo già che HC è la metà di AC in quanto nei triangoli isosceli l’altezza relativa alla base è anche mediana.
Proviamo ora l’implicazione opposta, ossia ABC è un triangolo rettangolo tale che l’altezza relativa all’ipotenusa sia esattamente la metà dell’ipotenusa allora ABC è anche isoscele. Se ci rifacciamo sempre all’immagine di prima abbiamo come ipotesi che \bar{BH}=\bar{HC}=\bar{HA} inoltre anche gli angoli BHC e BHA sono congruenti perchè BH è altezza, ne segue che i due triangoli BCH e BHA sono congruenti e pertanto i due angoli adiacenti all’ipotenusa di ABC sono congruenti da cui segue che ABC è isoscele.

Ritorna all’indice

Quesito 2

I modi di ottenere esattamente due testa in cinque lanci sono 10 (ad esempio TTCCC, TCTCC, TCCTC e così via), pertanto la probabilità di ottenere esattamente due teste è

    \[10\cdot p^2\cdot (1-p)^3\]

Per rispondere alla seconda domanda, siccome ci chiede un massimo, basta cercare i massimi della funzione probabilità, quindi come prima cosa la deriviamo in p e otteniamo

    \[10\cdot (2p(1-p)^3-3p^2(1-p)^2)=10p(1-p)^2(2-5p)\]

per cui la derivata ha tre zeri che sono p=0, p=1 e p=2/5. Le prime due soluzioni sono quelle che rendono nulla la probabilità che escano esattamente due teste, mentre la terza soluzione p=2/5 è quella che rende massima la probabilità che escano esattamente due teste.

Ritorna all’indice

Quesito 3

Ritorna all’indice

Quesito 4

Ritorna all’indice

Quesito 5


Ritorna all’indice

Quesito 6

Ritorna all’indice

Quesito 7

Ritorna all’indice

Quesito 8

Esame di maturità 2023 prova di matematica – Prova svolta

Indice

Problema 1

Punto a

Dal grafico presente nel testo dell’esercizio possiamo dedurre i punti da cui passano i tre pezzi di grafico (la pratica di dedurre i punti dal grafico non è completamente corretta, sarebbe stato meglio che il ministero avesse esplicitato i punti). Possiamo quindi dedurre che

il grafico \Gamma_1 parta dal punto A=(-2\:;\:0) e arrivi al punto B=(0\:;\:1), che il grafico \Gamma_2 parta dal punto B=(0\:;\:1) e arrivi al punto C=(1\:;\:0) e infine che il grafico \Gamma_3 parta dal punto C=(1\:;\:0). Una volta dedotto questo, per ottenere l’espressione analitica della funzione nell’intervallo [-2\:;\:2] ci basta sostituire, quindi imponendo il passaggio di \Gamma_1 per B avremo che

    \[y=a(x+2)^2\]

    \[1=a(0+2)^2\]

    \[1=4a\]

    \[a={1\over 4}\]

Imponendo il passaggio di \Gamma_2 sempre per B avremo che

    \[x^2+y^2+b=0\]

    \[0^2+1^2+b=0\]

    \[b=-1\]

e infine imponendo il passaggio di \Gamma_3 per C otteniamo

    \[x^2-y^2+c=0\]

    \[1^2+0^2+c=0\]

    \[c=-1\]

da cui avremo

    \[\begin{cases} y={1\over 4}(x+2)^2\qquad &se\:\in [-2\:;\:0] \\ x^2+y^2-1=0\: &se\:\in [0\:;\:1]\\ x^2-y^2-1=0\: &se\:\in [1\:;\:2] \end{cases} \Rightarrow y=f(x)=\begin{cases} {1\over 4}(x+2)^2\qquad &se\:\in [-2\:;\:0] \\\sqrt{1-x^2}\: &se\:\in [0\:;\:1]\\  \sqrt{x^2-1}\: &se\:\in [1\:;\:2] \end{cases}\]

Una volta dedotta la forma analitica della funzione possiamo osservare che all’interno dei tre intervalli [-2\:;\:0], [0\:;\:1] e [1\:;\:2] la funzione è sicuramente derivabile (questo perchè le funzioni definite nei singoli intervalli sappiamo essere derivabili), ci rimangono solo dei dubbi sui due punti di ascissa x=0 e x=1. Prima di procedere analiticamente con il calcolo ragioniamo sul concetto di derivata. Sappiamo che la derivata è il coefficiente angolare della retta tangente al grafico e sappiamo che questo coefficiente angolare deve essere un numero finito affinché la funzione sia derivabile. Pertanto in entrambi i punti la funzione non sarà derivabile, infatti nel punto di ascissa x=0 la retta tangente che si ottiene percorrendo il grafico \Gamma_1 verso destra (retta nera) è diversa dalla retta tangente che si ottiene percorrendo il grafico \Gamma_2 verso sinistra (retta blu) e quindi il punto è un punto angoloso

mentre il punto di ascissa x=1 possiede la retta tangente parallela all’asse y (retta verde) e quindi è un punto di cuspide

Una volta fatto questo ragionamento intuitivo supportiamo tale ragionamento attraverso i calcoli analitici delle derivate. Allora

    \[f'(x)=\begin{cases} {1\over 4}\cdot 2\cdot (x+2)={1\over 2}(x+2)\qquad &se\:\in [-2\:;\:0] \\\\{1\over 2}(1-x^2)^{-1/2}\cdot(-2x)={-x\over \sqrt{1-x^2}}\: &se\:\in [0\:;\:1]\\  \\{1\over 2}(x^2-1)^{-1/2}\cdot(2x)={x\over \sqrt{x^2-1}}\: &se\:\in [1\:;\:2] \end{cases}\]

Da cui è possibile vedere che le due derivate calcolate nel punto x=0 sono diverse a seconda di che espressione consideriamo e che la derivata nel punto x=1 ha valore non finito. A questo punto completiamo il punto a) del problema calcolando la derivata nei punti di ascissa x=-2, x=0, x=1 e x=2. La retta tangente al punto di ascissa x=-2 è la retta che passa per f(-2) e ha come coefficiente angolare f'(-2) (questo vale per ogni punto di derivabilità della funzione e quindi anche per x=2, mentre per gli altri due punti dovremo fare due discorsi differenti). Pertanto, siccome f(-2)=0 e f'(-2)=0, la retta tangente al grafico nel punto di ascissa x=-2 è y=0 che è esattamente l’asse x. La retta tangente al punto x=0, come abbiamo già ampiamente detto, non esiste. La retta tangente al punto di ascissa x=1 (la retta verde del grafico precedente) è la retta x=1. Infine la retta tangente al grafico nel punto x=2, siccome f(2)=\sqrt 3 e f'(2)={2\over \sqrt 3}, ha equazione

    \[y-\sqrt3={2\over \sqrt 3}\cdot (x-2)\]

    \[y={2\over \sqrt 3}x-{4\over \sqrt 3}+\sqrt 3\]

    \[y={2\over \sqrt 3}x+{-4+3\over \sqrt 3}\]

    \[y={2\over \sqrt 3}x-{1\over \sqrt 3}\]

Ritorna all’indice

Punto b

Per rispondere al punto b) osserviamo che dobbiamo dedurre il grafico della derivata a partire dal grafico della funzione f(x), non dobbiamo semplicemente dedurre il grafico di f'(x) partendo dalla sua espressione analitica (trovata nel punto a)). Nel primo tratto (quello relativo alla parabola) la derivata parte da un valore uguale a f'(-2)=0 e tende a f'(0)=1 (infatti abbiamo già detto nel punto x=0 la derivata non c’è, nonostante questo da sinistra la derivata tende ad essere 1). Nel secondo tratto (quello relativo alla circonferenza) la derivata parte da f'(0)=0 (ancora una volta il valore 0 non è considerato) e tende ad arrivare a -\infty (infatti la tangente tende alla retta x=1). Superato il punto x=1 e arrivati all’ultimo tratto (quello relativo al’arco di iperbole) si ha che la derivata parte da +\infty fino ad arrivare a f'(2)=2/\sqrt3. In definitiva il grafico sarà qualcosa del tipo

Cerchiamo ora di studiare la funzione integrale

    \[F(x)=\int_{-2}^{x}f(t)dt\]

siccome f(x) è continua nell’intervallo [-2\:;\:2] la prima parte del teorema fondamentale del calcolo integrale ci dice che F(x) è derivabile, inoltre F'(x)=f(x) e quindi

    \[F''(x)=f'(x)\]

pertanto per studiare la concavità e la convessità della funzione integrale ci basta studiare il grafico di f'(x) (che abbiamo appena dedotto). Per cui la funzione F(x) sarà convessa negli intervalli in cui F''(x)=f'(x) è positiva, ossia ]-2\:;\:0[ e ]1\:;\: 2[, e sarà concava negli intervalli in cui la derivata seconda è negativa, ossia ]0\:;\:1[.

Ritorna all’indice

Punto c

La funzione

    \[y={1\over 4}(x+2)^2\]

definita sull’intervallo [-2\:;\:0] è una funzione continua e strettamente crescente (possiamo vederlo graficamente oppure in maniera più precisa andando ad esaminare la derivata calcolata in precedenza che è sempre strettamente positiva tranne nel punto x=-2) pertanto sappiamo essere invertibile. Per calcolare la sua inversa ci basta determinare x in funzione di y, da cui

    \[y={1\over 4}(x+2)^2\]

    \[4y=(x+2)^2\]

    \[\sqrt{4y}=x+2\]

    \[x=\sqrt{4y}-2=2\sqrt y-2=2(\sqrt y-1)\]

Pertanto la funzione h(x) (quindi scritta cambiando il nome delle variabili) sarà

    \[h(x)=2(\sqrt x-1)\]

che è una funzione derivabile nel suo dominio naturale [0\:;\;+\infty[ e la cui derivata è

    \[h'(x)=2\cdot {1\over 2}\cdot {1\over \sqrt x}={1\over \sqrt x}\]

perfettamente definita nell’intervallo ]0\:;\:+\infty[. Infine per disegnarne il grafico osserviamo che

    \[y=2(\sqrt x-1)\Rightarrow x={1\over 4}(y+2)^2\]

pertanto il grafico di h(x) è uguale al grafico di f(x) in [-2\:;\:0] dove abbiamo invertito gli assi cartesiani (quindi sono speculari rispetto alla bisettrice del primo e terzo quadrante y=x), da cui

Dove abbiamo rappresentato solo la funzione h(x) inversa dell’arco di parabola di f(x).

Ritorna all’indice

Punto d

Risolviamo infine l’ultimo punto del primo problema. Cerchiamo di capire a livello grafico quello che dobbiamo fare: dobbiamo trovare la costante k per cui la retta x=k (retta verde) divide in due parti di equale area il sottografico di \Gamma_1

Quindi prima di tutto calcoliamo l’area del sottografico di \Gamma_1 e poi imponiamo che l’area del sottografico di \Gamma_1 fino a x=k sia esattamente la sua metà. Per fare questo utilizziamo il calcolo integrale (nonostante si possano usare anche le nozioni di geometria analitica imparate in terza per l’area del segmento parabolico). Allora

    \[A_{\Gamma_1}^{[-2\:;\:0]}=\int_{-2}^{0}{1\over 4}(x+2)^2dx={1\over 4}\cdot\Big[{1\over 3}(x+2)^3\Big]_{-2}^0={2\over 3}\]

da cui

    \[A_{\Gamma_1}^{[-2\:;\:k]}={1\over 3}\]

    \[\int_{-2}^{k}{1\over 4}(x+2)^2dx={1\over 3}\]

    \[{1\over 4}\cdot\Big[{1\over 3}(x+2)^3\Big]_{-2}^0={1\over 3}\]

    \[{1\over 12}(k+2)^3={1\over 3}\]

    \[(k+2)^3=4\]

    \[k+2=\sqrt[3]4\]

    \[k=\sqrt[3]4-2\]

Ritorna all’indice

Problema 2

Punto a

Studiando, al variare di a\neq 0, la funzione

    \[f_a(x)={x^2-ax\over x^2-a}\]

possiamo subito notare che il numeratore e il denominatore sono continui nel loro dominio naturale e che l’unico “problema” potrebbe essere rappresentato dal denominatore che dovrà essere sempre diverso da zero per dare senso alla divisione. Fatta questa premessa risulta evidente che se fosse a<0 allora -a>0 per cui x^2-a>0 e quindi la funzione sarebbe continua in tutto \mathbb{R}. Nel caso in cui a>0 dobbiamo imporre che il denominatore sia sempre diverso da zero, cioè

    \[x^2-a\neq 0\Rightarrow x\neq \pm\sqrt a\]

quindi abbiamo due punti di discontinuità in x=-\sqrt a e x=+\sqrt a. Per studiare che punti di discontinuità sono e i vari asintoti utilizziamo i limiti. Per a<1, siccome non abbiamo punti di discontinuità all’interno di \mathbb{R}, potremmo avere solo asintoti orizzontali o obliqui, pertanto studiamo

    \[\lim_{x\to -\infty}f(x)=\lim_{x\to -\infty}{x^2-ax\over x^2-a}=1\]

    \[\lim_{x\to +\infty}f(x)=\lim_{x\to +\infty}{x^2-ax\over x^2-a}=1\]

e questo a prescindere dal valore di a\neq 0. Pertanto in ogni caso la funzione f(x) presenta l’asintoto orizzontale y=1. Studiamo ora i limiti per x\to -\sqrt a e per x\to +\sqrt a nel caso in cui a>0. Allora se a\neq 1

    \[\lim_{x\to -\sqrt a^-}f(x)=\lim_{x\to -\sqrt a^-}{x^2-ax\over x^2-a}=+\infty\]

a prescindere dal valore di a>0\:,\:a\neq 1 in quanto il numeratore è sempre un numero positivo diverso da zero e il denominatore tende a 0^+. Un discorso analogo lo possiamo fare per

    \[\lim_{x\to -\sqrt a^+}f(x)=\lim_{x\to -\sqrt a^+}{x^2-ax\over x^2-a}=-\infty\]

dove osserviamo esplicitamente che tutto questo è così perchè se x\to-\sqrt a il numeratore x^2-ax tende sicuramente a qualcosa di positivo. Discorso diverso abbiamo nel limite per x\to +\sqrt a, infatti in questo caso il numeratore sarà una volta positivo e una volta negativo. In particolare

    \[\lim_{x\to +\sqrt a^-}f(x)=\lim_{x\to +\sqrt a^-}{x^2-ax\over x^2-a}=\pm\infty\]

infatti il denominatore tende sempre a 0^- a prescindere dal valore di a, mentre il numeratore tende ad essere un numero positivo se 0<a<1 e negativo se a>1. Pertanto tale limite sarà -\infty se 0<a<1 e +\infty se a>1. Discorso del tutto analogo (ma al contrario con i segni) possiamo farlo per

    \[\lim_{x\to +\sqrt a^+}f(x)=\lim_{x\to +\sqrt a^+}{x^2-ax\over x^2-a}=\mp\infty\]

Da questa discussione con i limiti viene fuori che il caso a=1 comporta delle leggere differenze, infatti a uno studio un attimo più approfondito si nota che quando a=1 allora

    \[f_1(x)={x^2-x\over x^2-1}={x\cdot (x+1)\over (x+1)\cdot (x-1)}={x\over x-1}\]

che quindi ha dominio \mathbb{R}-\{1\} e

    \[\lim_{x\to 1}f_1(x)=\lim_{x\to 1}{{x\over x-1}}=\mp\infty\]

negativo se x\to 1^- positivo se x\to 1^+. Ricapitolando abbiamo tutti punto di discontinuità di seconda specie e come asintoti quello orizzontale y=1 per ogni a\neq 0, quelli verticali x=\pm\sqrt a per ogni a>0 e a\neq 1 e invece per a=1 quello verticale x=1. Nel dettaglio le quattro tipologie di grafico di f_a(x) sono

Ritorna all’indice

Punto b

Le intersezioni tra \Omega_a e il suo asintoto orizzontale sono le soluzione del sistema

    \[\begin{cases} y={x^2-ax\over x^2-a} \\\\y=1 \end{cases}\]

che è facilmente risolvile per sostituzione

    \[\begin{cases} y={x^2-ax\over x^2-a} \\y=1 \end{cases}=\begin{cases} 1={x^2-ax\over x^2-a} \\y=1 \end{cases}=\begin{cases} x^2-a=x^2-ax \\y=1 \end{cases}=\begin{cases} -a=-ax \\y=1 \end{cases}=\begin{cases} x=1\\y=1 \end{cases}\]

quindi la soluzione è indipendente dal parametro a. Osserviamo anche esplicitamente che tale soluzione per a=1 non la possiamo accettare (nonostante quel sistema sia comunque risolvibile nello stesso identico modo) perchè il dominio di f_1(x) non contiene il punto x=1. Per rispondere al secondo punto facciamo vedere che f_a(0) e f'_a(0) sono indipendenti dal parametro a qualunque a\neq 1 si consideri. Infatti

    \[f_a(0)={0^2-a\cdot 0\over 0-a}=-{0\over a}=0\]

Invece

    \[f'_a(x)={(2x-a)(x^2-a)-(x^2-ax)\cdot 2x\over (x^2-a)^2}\]

    \[f'_a(x)={2x^3-2ax-ax^2+a^2-2x^3+2ax^2\over (x^2-a)^2}\]

    \[f'_a(x)={ax^2-2ax+a^2\over (x^2-a)^2}\]

pertanto

    \[f'_a(0)=1\]

qualunque sia il parametro a\neq 1.

Ritorna all’indice

Punto c

Per studiare la monotonia di una funzione dobbiamo studiare il segno della derivata prima di tale funzione. Dal punto a) sappiamo che

    \[f'_a(x)={ax^2-2ax+a^2\over (x^2-a)^2}\]

pertanto il denominatore è sempre positivo e il segno della derivata dipende esclusivamente dal segno del numeratore. Il numeratore è una parabola che tocca l’asse x nei punti che verificano l’equazione associata

    \[ax^2-2ax+a^2=0\]

quindi

    \[\Delta=4a^2-4a^3=4a^2\cdot(1-a)>0\:quando\:a\neq 0\:e\:a<1\]

    \[x_{1,2}={2a\pm\sqrt{4a^2\cdot(1-a)}\over 2a}={2a\pm 2a\cdot\sqrt{1-a}\over 2a}=1\pm\sqrt{1-a}\]

e siccome quando a<0 la parabola ha concavità rivolta verso il basso sappiamo che sarà negativa esternamente e positiva internamente e quindi il segno della derivata sarà

mentre quando 0<a<1 la parabola avrà concavità rivolta verso l’alto e quindi il suo segno sarà

Notiamo anche esplicitamente che questi segni sono il linea con i grafici che abbiamo ottenuto nel punto a).
Per completare il punto c) studiamo la funzione che si ottiene quando a=-1, ossia

    \[f_{-1}(x)={x^2+x\over x^2+1}\]

osserviamo che sappiamo già tutto di questa funzione, infatti nei punti precedenti ne abbiamo studiato il dominio, i limiti, la monotonia e i punti di massimo e minimo (deducibili dal grafico dei segni scritto sopra), pertanto possiamo concludere che il grafico \Omega_{-1} sarà

Ritorna all’indice

Punto d

Per risolvere questo ultimo punto è necessario prima di tutto capire perfettamente quale area ci viene richiesto di calcolare, pertanto preliminarmente calcoliamo la tangente al grafico nell’origine e vediamo come è posizionata rispetto al grafico \Omega_{-1}. Dal punto b) dell’esercizio sappiamo che la retta tangente è la bisettrice del primo e terzo quadrante, ossia y=x e questa retta interseca il grafico nei due punti x=0 e x=1, infatti queste sono le soluzioni del seguente sistema

    \[\begin{cases} y={x^2+x\over x^2+1} \\\\y=x \end{cases}\]

    \[\begin{cases} x={x^2+x\over x^2+1} \\\\y=x \end{cases}\]

    \[\begin{cases} (x^2+1)\cdot x=x^2+x \\\\y=x \end{cases}\]

    \[\begin{cases} x^3+x=x^2+x \\\\y=x \end{cases}\]

    \[\begin{cases} x^3-x^2=0 \\\\y=x \end{cases}\]

    \[\begin{cases} x^2\cdot (x-1)=0 \\\\y=x \end{cases}\]

Per concludere la nostra idea dell’area da calcolare (necessaria per imbastire correttamente l’integrale che ci permetterà di completare il calcolo) cerchiamo di capire se la retta è minore o maggiore di \Omega_{-1}. Per fare questo studiamo il segno della seguente funzione

    \[{x^2+x\over x^2+1}-x\]

che è la sottrazione tra la funzione f_{-1}(x) e la retta. Pertanto

    \[{x^2+x\over x^2+1}-x>0\]

    \[{x^2+x-x^3-x\over x^2+1}>0\]

    \[{x^2-x^3\over x^2+1}>0\]

    \[{x^2\cdot (1-x)\over x^2+1}>0\]

e quindi per studiare il segno complessivo della frazione studiamo il segno dei tre “blocchi” separatamente. Sappiamo che x^2 e x^2+1 sono sempre positivi, mentre 1-x>0 se x<1, per cui lo schema dei segni sarà

Da cui possiamo dedurre che il grafico \Omega_{-1} sta “sopra” la retta tra 0 e 1 e “sotto” tra 1 e +\infty. La situazione sarà qualcosa del tipo

Per calcolare l’area compresa tra le rette blu, rosse e il grafico verde impostiamo il seguente calcolo integrale

    \[\int_{1}^{\sqrt{3}}xdx-\int_{1}^{\sqrt{3}}{x^2+x\over x^2+1}dx\]

    \[\int_{1}^{\sqrt{3}}xdx-\int_{1}^{\sqrt{3}}{x^2+1-1+x\over x^2+1}dx\]

    \[\int_{1}^{\sqrt{3}}xdx-\int_{1}^{\sqrt{3}}{x^2+1\over x^2+1}+{-1+x\over x^2+1}dx\]

    \[\int_{1}^{\sqrt{3}}xdx-\int_{1}^{\sqrt{3}}1+{x\over x^2+1 }-{1\over x^2+1}dx\]

    \[\int_{1}^{\sqrt{3}}xdx-\int_{1}^{\sqrt{3}}1dx-\int_{1}^{\sqrt{3}}{x\over x^2+1}dx+\int_{1}^{\sqrt{3}}{1\over x^2+1}dx\]

    \[\Big[ {x^2\over 2}\Big]_{1}^{\sqrt{3}}-\Big[x \Big]_{1}^{\sqrt{3}}-\Big[{1\over 2 }\ln(x^2+1) \Big]_{1}^{\sqrt{3}}+\Big[\arctan(x) \Big]_{1}^{\sqrt{3}}\]

    \[({3\over 2}-{1\over 2})-(\sqrt 3-1)-({1\over 2}\cdot \ln(4)-{1\over 2}\cdot \ln(2))+(\arctan(\sqrt 3)-\arctan(1))\approx 0,18\]

Ritorna all’indice

Quesito 1

Prima di tutto costruiamo la figura passo passo. Quindi tracciamo il triangolo rettangolo

Tracciamo il quadrato di lato BC

e infine tracciamo le due distanze che dobbiamo dimostrare essere congruenti

A questo punto procediamo con la dimostrazione. L’idea è quella di dimostrare che i due triangoli BHO e GOC sono congruenti

Sicuramente, per definizione di distanza, sono due triangoli rettangoli, inoltre il lato CO e il lato BO sono congruenti in quanto mezza diagonale del quadrato. A questo punto se dimostriamo che gli angoli sono congruenti abbiamo terminato. Ma questo è relativamente facile da vedere, infatti l’angolo HBD e l’angolo ACB sono congruenti perchè entrambi sono calcolabili con la formula 180^\circ-90^\circ-ABC mentre gli angoli DBO e BCO sono congruenti (e pari a 45^\circ) in quanto la diagonale del quadrato è anche bisettrice degli angoli interni. Pertanto OH è congruente a OG.

Ritorna all’indice

Quesito 2

Come primo passo dobbiamo determinare le probabilità p e q che esca un numero pari e che esca un numero dispari. Sicuramente dal testo dell’esercizio sappiamo che p=2q. Inoltre, dalle definizioni classiche di probabilità, sappiamo che la somma delle probabilità di tutti gli eventi elementari deve essere 1, ossia 3p+3q=1. Pertanto risolvendo il sistema

    \[\begin{cases} p&=2q \\3p+3q&=1 \end{cases}\]

si ottengono facilmente i valori di p=2/9 e q=1/9. Una volta ottenute le due probabilità cercate ragioniamo sugli eventi proposti dal testo dell’esercizio. La probabilità che esca un numero primo è uguale alla probabilità che esca un numero tra 2, 3 e 5, per cui

    \[P(primo)=P(2)+P(3)+P(5)={2\over 9}+{1\over 9}+{1\over 9}={4\over 9}\]

La probabilità che esca un numero almeno pari a 3 è uguale alla probabilità che esca un numero tra 3, 4, 5 e 6, per cui

    \[P(almeno\:3)=P(3)+P(4)+P(5)+P(6)={1\over 9}+{2\over 9}+{1\over 9}+{2\over 9}={6\over 9}={2\over 3}\]

La probabilità che esca un numero al più pari a 3 è uguale alla probabilità che esca un numero tra 1, 2 e 3, per cui

    \[P(al\:più\:3)=P(1)+P(2)+P(3)={1\over 9}+{2\over 9}+{1\over 9}={4\over 9}\]

Ritorna all’indice

Quesito 3

Risolviamo il quesito come se fosse un problema di geometria analitica di terza, quindi per prima cosa determiniamo l’equazione della retta passante per A e B e in seguito cerchiamo, tra tutte le sfere di centro C, l’unica che interseca la retta in un unico punto. Sappiamo che i punti che appartengono alla retta passante per A e B sono i punti che si possono ottenere partendo dal punto A aggiungendo un vettore t\cdot \vec{AB} (equazione parametriche delle rette)

    \[(x\:;\:y\:;\:z)=(x_A\:;\:y_A\:;\:z_A)+t\cdot (x_B-x_A\:;\:y_B-y_A\:;\:z_B-z_A)\]

    \[(x\:;\:y\:;\:z)=(1\:;\:-2\:;\:0)+t\cdot (2-1\:;\:3-(-2)\:;\:-1-0)=(1+t\:;\:-2+5t\:;\:-t)\]

L’equazione della generica sfera di centro C e raggio incognito r è data da

    \[(x-x_C)^2+(y-y_C)^2+(z-z_C)^2=r^2\]

    \[(x-1)^2+(y+6)^2+(z-7)^2=r^2\]

Una volta determinato questo andremo a risolvere il sistema

    \[\begin{cases} (x\:;\:y\:;\:z)=(1+t\:;\:-2+5t\:;\:-t) \\(x-1)^2+(y+6)^2+(z-7)^2=r^2 \end{cases}\]

cercando il raggio r in maniera tale che il delta dell’equazione di secondo grado che risolve il sistema sia nullo. Pertanto per sostituzione otteniamo

    \[(1+t-1)^2+(-2+5t+6)^2+(-t-7)^2=r^2\]

    \[t^2+16+25t^2+40t+t^2+49+14t-r^2=0\]

    \[27t^2+54t+65-r^2\]

da cui

    \[\Delta=54^2-4\cdot 27\cdot (65-r^2)=108r^2-4104\]

e quindi

    \[\Delta=0\Rightarrow 108r^2-4104=0\Rightarrow r^2=38\]

Pertanto l’equazione della sfera sarà

    \[(x-1)^2+(y+6)^2+(z-7)^2=38\]

Ritorna all’indice

Quesito 4

Partiamo dalla prima parte del quesito e cerchiamo di capire che caratteristiche hanno i parallelepipedi rettangoli a base quadrata di volume costante V. Se di tale parallelepipedo chiamiamo l il lato di base e h l’altezza possiamo senza dubbio dire che

    \[A=2l^2+4lh\]

    \[V=l^2h\]

da cui

    \[h={V\over l^2}\]

    \[A=2l^2+4l\cdot {V\over l^2}=2l^2+{4V\over l}\]

Una volta determinata l’espressione analitica dell’area totale che dipende dall’unico parametro l è possibile minimizzare la funzione e determinare che caratteristiche deve avere l per far si che l’area totale sia la minima possibile. Quindi partiamo da A(l) e calcoliamo la sua derivata prima rispetto a l, ossia

    \[A'(l)=4l-{4V\over l^2}\]

da cui

    \[A'(l)=0\iff 4l-{4V\over l^2}=0\]

    \[4l={4V\over l^2}\]

    \[l^3=V\]

    \[l=\sqrt[3]{V}\]

e studiando il segno della derivata prima possiamo dedurre che questo è un punto di minimo della nostra funzione A(l). Per completare il quesito ci basta verificare quale sia la condizione di lunghezza minima della diagonale. Sappiamo che

    \[d=\sqrt{l^2+l^2+h^2}=\sqrt{2l^2+{V^2\over l^4}}=\Big(2l^2+{V^2\over l^4}\Big)^{1\over 2}\]

per cui la derivata della funzione d(l) sarà

    \[d'(l)={1\over 2}\cdot \Big(2l^2+V^2l^{-4}\Big)^{-{1\over 2}}\cdot \Big(4l+{(-4)V^2\over l^5}\Big)={2l-{2V^2\over l^5}\over \sqrt{2l^2+{V^2\over l^4}}}\]

da cui, considerando che il denominatore è sempre maggiore di zero, avremo che

    \[d'(l)=0\iff 2l-{2V^2\over l^5}=0\]

    \[2l={2V^2\over l^5}\]

    \[l^6=V^2\]

    \[l=\sqrt[3]V\]

Quindi la condizione che rende minima l’area totale è la stessa che rende minima la diagonale.

Ritorna all’indice

Quesito 5

Come due metodi proponiamo quello con il calcolo della derivata e quello che impone che l’intersezione tra retta e curva abbiamo un’unica soluzione doppia. La curva definita dalla funzione

    \[y=\sqrt{25-x^2}\]

è la semicirconferenza di centro (0\:;\:0) e raggio r=5, inoltre passa dal punto (3\:;\:\sqrt{25-3^2})=(3\:;\:4). Una volta dedotto questo partiamo con il primo metodo.

    \[y'={-x\over \sqrt{25-x^2}}\]

per cui y'(3)=-3/4. Quindi la retta tangente alla curva è l’unica retta passante per (3\:;\:4) e avente coefficiente angolare -3/4, ossia

    \[y-4=-{3\over 4}\cdot (x-3)\Rightarrow y=-{3\over 4}x+{25\over 4}\]

Il secondo metodo è quello studiato in terza che utilizza le conoscenze di geometria analitica. Quindi prima di tutto intersechiamo la generica retta passante per (3\:;\:4) con la circonferenza, ossia

    \[\begin{cases} x^2+y^2=25 \\y-4=m\cdot (x-3) \end{cases}\]

    \[\begin{cases} x^2+y^2=25 \\y=mx -3m+4 \end{cases}\]

    \[\begin{cases} x^2+(mx -3m+4)^2=25 \\y=mx -3m+4 \end{cases}\]

    \[\begin{cases} x^2+m^2x^2+9m^2+16-6m^2x+8mx-24m-25=0 \\y=mx -3m+4 \end{cases}\]

    \[\begin{cases} (1+m^2)x^2+(8m-6m^2)x+(9m^2-24m-9)=0 \\y=mx -3m+4 \end{cases}\]

a questo punto determiniamo m in maniera tale che il sistema abbia un’unica soluzione, ossia che \Delta_x =0, allora

    \[\Delta_x =0\iff (8m-6m^2)^2-4\cdot(1+m^2)\cdot(9m^2-24m-9)=0\]

    \[64m^2+36m^4-96m^3-36m^2+96m+36-36m^4+96m^3+36m^2=0\]

    \[64m^2+96m+36=0\]

    \[\Delta_m=96^2-4\cdot 64\cdot 36=9216-9216=0\]

    \[m_1=m_2={-96\pm 0\over 128}=-{3\over 4}\]

Per cui la retta tangente è la stessa di prima, ossia y=-{3\over 4}x+{25\over 4}

Ritorna all’indice

Quesito 6

Se proviamo a calcolare

    \[\lim_{x\to 0}{\sin x-(ax^3+bx)\over x^3}\]

ci accorgiamo subito che siamo di fronte a una forma indeterminata del tipo 0/0 e quindi, viste anche le caratteristiche della funzione, siamo dentro le ipotesi del teorema di de l’Hopital. Per cui

    \[\lim_{x\to 0}{\sin x-(ax^3+bx)\over x^3}=\lim_{x\to 0}{\cos x-(3ax^2+b)\over 3x^2}\]

questo secondo limite ha il numeratore che tende a 1-b mentre il denominatore tende a 0, pertanto se vogliamo che il limite complessivo tendi a 1 abbiamo come unica soluzione che anche il numeratore tenda a 0 (infatti se sopra ci fosse un qualunque numero diverso da 0 oppure \pm\infty il limite sarebbe \pm\infty). Da questa osservazione possiamo capire che 1-b=0\Rightarrow b=1. Una volta determinato questo, per quello che abbiamo già detto, il nostro limite ottenuto tramite de l’Hopital verifica ancora le ipotesi del teorema (per b=1), pertanto

    \[\lim_{x\to 0}{\sin x-(ax^3+x)\over x^3}=\lim_{x\to 0}{\cos x-(3ax^2+1)\over 3x^2}=\lim_{x\to 0}{-\sin x-6ax\over 6x}=-{1\over 6}-a\]

quindi abbiamo

    \[-{1\over 6}-a=1\Rightarrow a=-{7\over 6}\]

Ritorna all’indice

Quesito 7

La funzione definita a tratti

    \[f(x)=\begin{cases} -1+\arctan(x) &se\:x<0 \\ax+b &se\:x\ge 0 \end{cases}\]

è una funzione derivabile in ogni suo parte quindi per determinare se è derivabile complessivamente bisogna capire cosa succede per x=0. La funzione derivata f'(x) è la funzione

    \[f'(x)=\begin{cases} {1\over 1+x^2} &se\:x<0 \\a &se\:x> 0 \end{cases}\]

Per determinare per quali valori di a e b la funzione f(x) è derivabile prima di tutto determiniamo le condizioni affinchè f(x) è continua in x=0 e in seguito quella di derivabilità. f(x) è continua in x=0 se

    \[\lim_{x\to 0^-}f(x)=\lim_{x\to 0^+}f(x)\]

    \[\lim_{x\to 0^-}-1+\arctan(x)=\lim_{x\to 0^+}ax+b\]

    \[-1=b\]

Una volta determinato b=-1 determiniamo la condizione di derivabilità, ossia

    \[\lim_{x\to 0^-}f'(x)=\lim_{x\to 0^+}f'(x)\]

    \[\lim_{x\to 0^-}{1\over 1+x^2} =\lim_{x\to 0^+}a\]

    \[1=a\]

Terminiamo l’esercizio rispondendo alla domanda sul teorema di Rolle. Le ipotesi del teorema sono: continuità, derivabilità è l’esistenza di due punti x_1 e x_2 tali che f(x_1)=f(x_2). Abbiamo già provato le prime due condizioni, verifichiamo se è possibile che si verifichi la terza. Osservando la derivata f'(x) notiamo che questa è sempre strettamente positiva, ne segue che la funzione f(x) è strettamente crescente e pertanto il teorema di Rolle non può mai verificarsi.

Ritorna all’indice

Quesito 8

La funzione f_a(x)=x^5-5ax+a con a>0 è una funzione continua e derivabile in tutto R pertanto, dal teorema di Bolzano, possiamo stabilire che f_a(x) con a>0 ammette sempre almeno uno zero. Infatti

    \[lim_{x\to -\infty}f_a(x)=-\infty\:mentre\:lim_{x\to +\infty}f_a(x)=+\infty\]

La funzione f_a(x) quindi ammetterà tre zeri se partendo da meno infinito passa nella parte positiva delle y poi scende di nuovo nella parte negativa e infine va verso più infinto, insomma se il suo andamento è quello rappresentato da questo grafico

Questa condizione, a livello matematico, può essere espressa dicendo che il massimo relativo della funzione deve essere positivo e il minimo relativo della funzione deve essere negativo, e quindi

    \[f_a^{max}\cdot f_a^{min}<0\]

Una volta osservato questo andiamo a risolvere l’esercizio. Per prima cosa determiniamo l’espressione del massimo e del minimo relativo sapendo che i due punti annullano la derivata prima, pertanto

    \[f'_a(x)=0\iff 5x^4-5a=0\:con\: a>0\iff x=\pm\sqrt[4]a\]

pertanto i punti di massimo e minimo hanno valori

    \[f_a(-\sqrt[4]a)=(-\sqrt[4]a)^5-5a(-\sqrt[4]a)+a=-\sqrt[4]{a^5}+5a\sqrt[4]a+a=a+4\sqrt[4]a\]

e analogamente il minimo è

    \[f_a(+\sqrt[4]a)=a-4\sqrt[4]a\]

da cui

    \[(a+\sqrt[4]a)\cdot (a-\sqrt[4]a)<0\:con\:a>0\]

    \[a^2-16a^2\sqrt a<0\:con\:a>0\]

    \[a^2\cdot (1-16\sqrt a)<0\:con\:a>0\]

    \[1-16\sqrt a<0\:con\:a>0\]

    \[a>{1\over\sqrt {256}}\]

Esame di maturità scientifico 2024 – prove svolte e simulazioni

Firmato dal ministro Valditara il decreto che sancisce la materia della seconda prova nei licei scientifici ossia MATEMATICA (sia per per gli indirizzi sportivi che scienze applicate). Alcune prove sono svolte e risolte, di altre invece ci sono solo i testi. Inoltre riportiamo le notizie aggiornate sull’esame di maturità 2024

MatematicaAnno 2024File pdfProva svolta
MatematicaAnno 2023File pdfProva svolta
Matematica e fisica Anno 2019File pdf
MatematicaAnno 2018File pdf
MatematicaAnno 2017File pdf
MatematicaAnno 2016File pdf
MatematicaAnno 2015File pdf
Matematica liceo ordinarioAnno 2014File pdf
Matematica liceo sperimentaleAnno 2014File pdf
Matematica liceo ordinario Anno 2013 File pdf
Matematica liceo sperimentale Anno 2013 File pdf
Matematica liceo ordinario Anno 2012 File pdf
Matematica liceo sperimentale Anno 2012 File pdf
Matematica liceo ordinario Anno 2011 File pdf
Matematica liceo sperimentale Anno 2011 File pdf
Matematica liceo ordinario Anno 2010 File pdf
Matematica liceo sperimentale Anno 2010 File pdf
Matematica liceo ordinario Anno 2009 File pdf
Matematica liceo sperimentale Anno 2009 File pdf
Matematica liceo ordinario Anno 2008 File pdf
Matematica liceo sperimentale Anno 2008 File pdf
Matematica liceo ordinario Anno 2007 File pdf
Matematica liceo sperimentale Anno 2007 File pdf
Matematica liceo ordinario Anno 2006 File pdf
Matematica liceo sperimentale Anno 2006 File pdf
atematica liceo ordinario Anno 2005 File pdf
Matematica liceo sperimentale Anno 2005 File pdf